Let \(W\) be the subspace spanned by the vectors \[ \begin{bmatrix} 0 \\ -2 \\ -6 \\ -4 \end{bmatrix}, \qquad \begin{bmatrix} -5 \\ -3 \\ -34 \\ -26 \end{bmatrix}.\] Find a basis for the orthogonal complement \(W^\perp\) of \(W\).

Click for answer

Click for Latex code